You are on page 1of 17

Arzela-Ascoli Theorem

MOUMITA JANA

2022MAS1011
Instructor : Dr. G S Raju

Indian Institute Of Technology Ropar

22th September,2023
DEFINITION

Bounded Set:Let (X , d) be any metric space.A set D ⊆ X is said


to be bounded if ∃ 0 < M < ∞ and y ∈ X ∋ d(x, y ) < M ∀x ∈ D.

Totally Bounded Set:Let (x, d) be a metric space.A set D ⊂ X is


said to be totally bounded if ∀ϵ > 0 ∃ a finite -ϵ net for D , i,e
there exist a finite set of points x1 , x2 , x3 , ...., xn ∈ X such that
n
S
D⊆ Bϵ (xi ) where Bϵ (xi ) := {x ∈ X : d(x, xi ) < ϵ} denotes the
i=1
open-ϵ ball of xi in X .
Example:
I = (0, 1) ⊂ R is totally bounded.
Take ϵ > 0
Bϵ (0) = {x ∈ R : d(x, 0) < ϵ} = (−ϵ, ϵ)
Similarly we can construct Bϵ (ϵ), Bϵ (2ϵ)
Now , by Archimedean property of R , ∃n ∈ N s.t nϵ > 1
i,e after finite number of such step ,1 can be crossed .
n−1
S
Now (0, 1) ⊂ Bϵ (iϵ) = (−ϵ, nϵ) as ϵ > 0, nϵ > 1
i=0
∴ (0, 1) can be covered by a total no of n-open ϵ balls.
Note
Every finite set is totally bounded.

Theorem
A totally bounded set is always bounded.

Remark
A bounded set may not be totally bounded.
Counter Example :Let (l 2 , d) be an metric space. where ,

 
l 2 = xn = (xi )i∈N |( |xi |2 < ∞) and
P
i=1
n 1
|xi − yi |2 ) 2 ∀xn , yn ∈ l 2 .
P
d(xn , yn ) = (
i=1
 A = {e1 , e2 , e3 , ...} =
Let


2
P
en = (0, 0, ...1, 0, ....)|n ∈ N| |ei | = 1 < ∞
i=1
∴ A ⊂ l 2.
claim: A is bounded but not totally bounded.
Let ei , ej ∈ A, for any i, j ∈ N.
 √
̸ j
2 , ifi =
d(ei , ej ) =
0 , ifi = j

sup {d(ei , ej )|∀i, j ∈ N} = 2
∴ A is bounded.
Now, take ϵ = 12 ,then
1

B 1 (ei ) = en ∈ A|d(en , ei ) < 2 = ei
2
1
The only non-empty subset of A with diameter < 2 are singleton
element.
n
S
since A is infinite so,A ̸⊆ B 1 (ei ).
2
i=1
hence A is not totally bounded.
EQUICONTINUOUS: Let (X , d) be any metric space and
D ⊆ X . A family F of functions defined on D is said to be
equicontinuous if ∀ϵ > 0, ∃δ > 0 ∋ ∀f ∈ F, |f (x) − f (y )| < ϵ
whenever d(x, y ) < δ; x, y ∈ D
sin(nx)
Example: Let fn (x) = n2
is equicontinuous on all R
Let ϵ > 0,
|fn (x) − fn (y )| = | sin(nx)
n2
− sin(ny )
n2
|
1
= n2
| sin(nx) − sin(ny )|
1
Ry
= n 2 | x n cos(nt)dt|
1
R y
≤ n x | cos(nt)|dt
1
≤ n |y − x| < ϵ when d(x, y ) = |y − x| < nϵ = δ.
Theorem
1 Every member of equicontinuous family is uniformly
continuous.

2 Let (X , d) be a metric space and D ⊆ X Let F be a finite


collection of function from D to X .If each member of F is
uniformly continuous then F is equicontinuous .
Remark
This property may not hold for an infinite collection of functions.

Counter Example: Let fn (x) = sin(nx)n∈N , x ∈ [0, 2π]


clearly,fn (x) is uniformly continuous on [0, 2π].
claim:fn (x) is not equicontinuous
1
choose ϵ = 2
π
take x = 2n and y = 0
π
it is possible to choose n large enough that x = 2n < δ.
π
For that x and y , |x − y | = 2n < δ but
π 1
|fn (x) − fn (y )| = sin(n 2n )=1≮ 2
∴ fn (x) is not equicontinuous .
THEOREM
Let K be a compact metric space.Let fn ∈ c(k), n ∈ N.if
F = {fn }n∈N converges uniformly then the family F is
equicontinuous .
Proof: Let K be a compact metric space.
Let fn uniformly converges to f
Then by Cauchy criteria,
ϵ
∀ϵ > 0∃N ∈ N ∋ |fn (x) − fm (x)| < 3 , ∀n, m ≥ N
since each fn is continuous
for this ϵ > 0∃ δi > 0 ∋
ϵ
|fi (x) − fi (y )| < 3 when d(x, y ) < δi and i = 1, ..., N
Let δ = min {δ1 , δ2 , ....., δN }
For n > N, if d(x, y ) < δ then ,
|fn (x) − fn (y )| ≤ |fn (x) − fm (x)| + |fm (x) − fm (y )| + |fm (y ) − fn (y )|
< 3ϵ + ϵ
3 + ϵ
3

hence F is equicontinuous .
Arzela-Ascoli Theorem
Let C [0, 1] be the space of all continuous functions from [0, 1] to
R with supremum distance.Let F ⊆ C [0, 1] be non-empty. Then F
is compact iff F is closed,bounded and equicontinuous .
Proof: Let F is compact
⇒ Every sequence has a convergent subsequence.
⇒ Every cauchy sequence converges to the limit of subsequence .
⇒ F is complete.
⇒ F is closed .
Fix ϵ > 0, Let {Bϵ (f ); f ∈ F } is open cover of F .
Since , F is compact .
n
S
So, ∃f1 , ......, fn ∈ F ∋ F ⊆ Bϵ (fi ).
i=1
∴ F is totally bounded ⇒ F is bounded.
Claim:F is equicontinuous.
Construct an open cover of F .
ϵ

Uf = B 3ϵ (f ) = g ∈ [0, 1] : d(f , g ) < 3
since F compact
n
S
∃ a finite set G = {f1 , ......, fn } ∋ F ⊂ Ufi
i=1
i,e if f ∈ F ⇒ f ∈ Ufk for some fk ∈ G
ϵ ϵ
i,e d(f , fk ) < 3 ⇒ |f (x) − fk (x)| < 3
since G is finite ∴ G is equicontinuous.
Choose f ∈ F and x, y ∈ [0, 1] ∋ d(x, y ) < δ
|f (x) − f (y )| ≤ |f (x) − fk (x)| + |fk (x) − fk (y )| + |fk (y ) − f (y )| < ϵ
∴ F is equicontinuous .
Converse Part : Let F be equicontinuous
ϵ
Fix ϵ > 0 ,∃δ > 0 ∋ ∀f ∈ F , |f (x) − f (y )| < 3
whenever d(x, y ) < δ, x, y ∈ [0, 1]
since [0, 1] is compact in R ∴ totally bounded.
m
S
i,e ∃ x1 , ....., xm ∈ [0, 1] ∋ [0, 1] ⊂ Bδ (xi )
i=1
Define a map g : F → Rn ∋
g (f ) = fˆ where f ∈ F
and fˆ = (f (x1 ), f (x2 ), ......., f (xn ))
n o
Denote F̂ = fˆ : f ∈ F ⊆ Rn

clearly,F̂ is closed and bounded.


since F̂ is closed and bounded in Rn
∴ F̂ is compact.
⇒ F̂ is totally bounded.
m
i,e ϵ > 0 ∃ fˆ1 , ......fˆm ∈ F̂ s.t F̂ ⊂ B 3ϵ (fˆj )
S
j=1

Let f ∈ F ∃ j ∈ {1, 2....., m} ∋ ||fˆ − fˆj ||∞ < ϵ


3
Let x ∈ [0, 1] ∃ k ∈ {1, 2....., m} ∋ d(x, xk ) < δ
then, |f (x) − f (xk )| < 3ϵ , ∀f ∈ F
Now,
|f (x) − fj (x)| ≤ |f (x) − f (xk )| + |f (xk ) − fj (xk )| + |fj (xk ) − fj (x)|
ϵ ϵ ϵ
< 3 + 3 + 3 < ϵ.
m
S
∴f ∈ Bϵ (fj ) ∀f ∈ F
j=1

Hence F is compact.
THANK YOU

You might also like